Let $pgt 3$ be an prime. If $sum_{k=1}^{p-1} frac{1}{k}=frac{a}{b}$, where $gcd(a,b)=1$. Prove that $pmid a$....












0












$begingroup$



This question already has an answer here:




  • Proof that $sumlimits_i frac{(p-1)!}i$ is divisible by $p$

    2 answers





Let $pgt 3$ be an prime. Suppose $$sum_{k=1}^{p-1} frac{1}{k}=frac{a}{b}$$ where $gcd(a,b)=1$. Prove that $a$ is divisible by $p$.




Please give me some hint. Sorry for this types of writing. I am not familiar with this.










share|cite|improve this question











$endgroup$



marked as duplicate by rtybase, Namaste, Davide Giraudo, Cesareo, Leucippus Jan 3 at 0:42


This question has been asked before and already has an answer. If those answers do not fully address your question, please ask a new question.














  • 1




    $begingroup$
    Related: Wolstenholme's Theorem
    $endgroup$
    – TheSimpliFire
    Jan 2 at 9:04






  • 3




    $begingroup$
    HINT: working $pmod p$ you have $$sum_{k=1}^{p-1} k^{-1} = sum_{k=1}^{p-1} k equiv frac{p-1}{2} cdot p equiv 0$$ This makes sense because every integer $1 le k le p-1$ has a unique inverse $pmod p$ in that range.
    $endgroup$
    – Crostul
    Jan 2 at 9:29












  • $begingroup$
    Please check your hint .....
    $endgroup$
    – Supriyo Banerjee
    Jan 2 at 9:33










  • $begingroup$
    @Crostul you may have to be careful working mod $p$ though:..for example for some positive integer $n$ the equation $2^{-n} equiv_p 1$, but $2^{-n} not = 1$ for any positive integer $n$.
    $endgroup$
    – Mike
    Jan 2 at 17:13


















0












$begingroup$



This question already has an answer here:




  • Proof that $sumlimits_i frac{(p-1)!}i$ is divisible by $p$

    2 answers





Let $pgt 3$ be an prime. Suppose $$sum_{k=1}^{p-1} frac{1}{k}=frac{a}{b}$$ where $gcd(a,b)=1$. Prove that $a$ is divisible by $p$.




Please give me some hint. Sorry for this types of writing. I am not familiar with this.










share|cite|improve this question











$endgroup$



marked as duplicate by rtybase, Namaste, Davide Giraudo, Cesareo, Leucippus Jan 3 at 0:42


This question has been asked before and already has an answer. If those answers do not fully address your question, please ask a new question.














  • 1




    $begingroup$
    Related: Wolstenholme's Theorem
    $endgroup$
    – TheSimpliFire
    Jan 2 at 9:04






  • 3




    $begingroup$
    HINT: working $pmod p$ you have $$sum_{k=1}^{p-1} k^{-1} = sum_{k=1}^{p-1} k equiv frac{p-1}{2} cdot p equiv 0$$ This makes sense because every integer $1 le k le p-1$ has a unique inverse $pmod p$ in that range.
    $endgroup$
    – Crostul
    Jan 2 at 9:29












  • $begingroup$
    Please check your hint .....
    $endgroup$
    – Supriyo Banerjee
    Jan 2 at 9:33










  • $begingroup$
    @Crostul you may have to be careful working mod $p$ though:..for example for some positive integer $n$ the equation $2^{-n} equiv_p 1$, but $2^{-n} not = 1$ for any positive integer $n$.
    $endgroup$
    – Mike
    Jan 2 at 17:13
















0












0








0





$begingroup$



This question already has an answer here:




  • Proof that $sumlimits_i frac{(p-1)!}i$ is divisible by $p$

    2 answers





Let $pgt 3$ be an prime. Suppose $$sum_{k=1}^{p-1} frac{1}{k}=frac{a}{b}$$ where $gcd(a,b)=1$. Prove that $a$ is divisible by $p$.




Please give me some hint. Sorry for this types of writing. I am not familiar with this.










share|cite|improve this question











$endgroup$





This question already has an answer here:




  • Proof that $sumlimits_i frac{(p-1)!}i$ is divisible by $p$

    2 answers





Let $pgt 3$ be an prime. Suppose $$sum_{k=1}^{p-1} frac{1}{k}=frac{a}{b}$$ where $gcd(a,b)=1$. Prove that $a$ is divisible by $p$.




Please give me some hint. Sorry for this types of writing. I am not familiar with this.





This question already has an answer here:




  • Proof that $sumlimits_i frac{(p-1)!}i$ is divisible by $p$

    2 answers








elementary-number-theory prime-numbers divisibility






share|cite|improve this question















share|cite|improve this question













share|cite|improve this question




share|cite|improve this question








edited Jan 2 at 21:21









Namaste

1




1










asked Jan 2 at 8:31









Supriyo BanerjeeSupriyo Banerjee

1056




1056




marked as duplicate by rtybase, Namaste, Davide Giraudo, Cesareo, Leucippus Jan 3 at 0:42


This question has been asked before and already has an answer. If those answers do not fully address your question, please ask a new question.









marked as duplicate by rtybase, Namaste, Davide Giraudo, Cesareo, Leucippus Jan 3 at 0:42


This question has been asked before and already has an answer. If those answers do not fully address your question, please ask a new question.










  • 1




    $begingroup$
    Related: Wolstenholme's Theorem
    $endgroup$
    – TheSimpliFire
    Jan 2 at 9:04






  • 3




    $begingroup$
    HINT: working $pmod p$ you have $$sum_{k=1}^{p-1} k^{-1} = sum_{k=1}^{p-1} k equiv frac{p-1}{2} cdot p equiv 0$$ This makes sense because every integer $1 le k le p-1$ has a unique inverse $pmod p$ in that range.
    $endgroup$
    – Crostul
    Jan 2 at 9:29












  • $begingroup$
    Please check your hint .....
    $endgroup$
    – Supriyo Banerjee
    Jan 2 at 9:33










  • $begingroup$
    @Crostul you may have to be careful working mod $p$ though:..for example for some positive integer $n$ the equation $2^{-n} equiv_p 1$, but $2^{-n} not = 1$ for any positive integer $n$.
    $endgroup$
    – Mike
    Jan 2 at 17:13
















  • 1




    $begingroup$
    Related: Wolstenholme's Theorem
    $endgroup$
    – TheSimpliFire
    Jan 2 at 9:04






  • 3




    $begingroup$
    HINT: working $pmod p$ you have $$sum_{k=1}^{p-1} k^{-1} = sum_{k=1}^{p-1} k equiv frac{p-1}{2} cdot p equiv 0$$ This makes sense because every integer $1 le k le p-1$ has a unique inverse $pmod p$ in that range.
    $endgroup$
    – Crostul
    Jan 2 at 9:29












  • $begingroup$
    Please check your hint .....
    $endgroup$
    – Supriyo Banerjee
    Jan 2 at 9:33










  • $begingroup$
    @Crostul you may have to be careful working mod $p$ though:..for example for some positive integer $n$ the equation $2^{-n} equiv_p 1$, but $2^{-n} not = 1$ for any positive integer $n$.
    $endgroup$
    – Mike
    Jan 2 at 17:13










1




1




$begingroup$
Related: Wolstenholme's Theorem
$endgroup$
– TheSimpliFire
Jan 2 at 9:04




$begingroup$
Related: Wolstenholme's Theorem
$endgroup$
– TheSimpliFire
Jan 2 at 9:04




3




3




$begingroup$
HINT: working $pmod p$ you have $$sum_{k=1}^{p-1} k^{-1} = sum_{k=1}^{p-1} k equiv frac{p-1}{2} cdot p equiv 0$$ This makes sense because every integer $1 le k le p-1$ has a unique inverse $pmod p$ in that range.
$endgroup$
– Crostul
Jan 2 at 9:29






$begingroup$
HINT: working $pmod p$ you have $$sum_{k=1}^{p-1} k^{-1} = sum_{k=1}^{p-1} k equiv frac{p-1}{2} cdot p equiv 0$$ This makes sense because every integer $1 le k le p-1$ has a unique inverse $pmod p$ in that range.
$endgroup$
– Crostul
Jan 2 at 9:29














$begingroup$
Please check your hint .....
$endgroup$
– Supriyo Banerjee
Jan 2 at 9:33




$begingroup$
Please check your hint .....
$endgroup$
– Supriyo Banerjee
Jan 2 at 9:33












$begingroup$
@Crostul you may have to be careful working mod $p$ though:..for example for some positive integer $n$ the equation $2^{-n} equiv_p 1$, but $2^{-n} not = 1$ for any positive integer $n$.
$endgroup$
– Mike
Jan 2 at 17:13






$begingroup$
@Crostul you may have to be careful working mod $p$ though:..for example for some positive integer $n$ the equation $2^{-n} equiv_p 1$, but $2^{-n} not = 1$ for any positive integer $n$.
$endgroup$
– Mike
Jan 2 at 17:13












2 Answers
2






active

oldest

votes


















0












$begingroup$

Well



$$sum_{k=1}^{p-1} frac{1}{k} = sum_{k=1}^{p-1} frac{frac{(p-1)!}{k}}{(p-1)!}$$



Now $frac{(p-1)!}{k}$ is an integer for each such $k$, so writing $A=(p-1)!$, we note that $frac{A}{k} equiv_p (A mod p)(k^{-1} mod p)$. Thus



$$sum_{k=1}^{p-1} frac{(p-1)!}{k} doteq sum_{k=1}^{p-1} frac{A}{k} equiv_p sum_{k=1}^{p-1} (A mod p)(k^{-1} mod p).$$



WE then use the fact that each element $k$ has a unqiue inverse to conclude



$$sum_{k=1}^{p-1} (A mod p)(k^{-1} mod p) equiv_p sum_{k=1}^{p-1} (A mod p)k.$$



However, one can check that for a prime $p geq 3$ the following holds: $sum_{k=1}^{p-1} (k mod p) equiv_p 0$, concluding



$$sum_{k=1}^{p-1} (A mod p)(k^{-1} mod p) equiv_p sum_{k=1}^{p-1} (A mod p)k equiv_p 0,$$



which gives you what you need to show [make sure you see why, it follows because $p$ does not divide $(p-1)!$].






share|cite|improve this answer











$endgroup$





















    0












    $begingroup$

    For $kleq {p-1over 2}$ let $q_k := {1over k(p-k)}$



    So we have $$({1over 1}+{1over p-1})+({1over 2}+{1over p-2})+...+({1over {p-1over 2}}+{1over {p+1over 2}}) = {aover b}$$



    $$pq_1+pq_2+...+pq_{p-1over 2} = {aover b}$$



    Let $$q_1+q_2+...+q_{p-1over 2} = {cover (p-1)!}$$ for some integer $c$. So we have $$pcdot c cdot b = acdot (p-1)!implies pmid acdot (p-1)!implies pmid a$$






    share|cite|improve this answer









    $endgroup$













    • $begingroup$
      Is this not enough answer to your question?
      $endgroup$
      – Maria Mazur
      Jan 8 at 18:46


















    2 Answers
    2






    active

    oldest

    votes








    2 Answers
    2






    active

    oldest

    votes









    active

    oldest

    votes






    active

    oldest

    votes









    0












    $begingroup$

    Well



    $$sum_{k=1}^{p-1} frac{1}{k} = sum_{k=1}^{p-1} frac{frac{(p-1)!}{k}}{(p-1)!}$$



    Now $frac{(p-1)!}{k}$ is an integer for each such $k$, so writing $A=(p-1)!$, we note that $frac{A}{k} equiv_p (A mod p)(k^{-1} mod p)$. Thus



    $$sum_{k=1}^{p-1} frac{(p-1)!}{k} doteq sum_{k=1}^{p-1} frac{A}{k} equiv_p sum_{k=1}^{p-1} (A mod p)(k^{-1} mod p).$$



    WE then use the fact that each element $k$ has a unqiue inverse to conclude



    $$sum_{k=1}^{p-1} (A mod p)(k^{-1} mod p) equiv_p sum_{k=1}^{p-1} (A mod p)k.$$



    However, one can check that for a prime $p geq 3$ the following holds: $sum_{k=1}^{p-1} (k mod p) equiv_p 0$, concluding



    $$sum_{k=1}^{p-1} (A mod p)(k^{-1} mod p) equiv_p sum_{k=1}^{p-1} (A mod p)k equiv_p 0,$$



    which gives you what you need to show [make sure you see why, it follows because $p$ does not divide $(p-1)!$].






    share|cite|improve this answer











    $endgroup$


















      0












      $begingroup$

      Well



      $$sum_{k=1}^{p-1} frac{1}{k} = sum_{k=1}^{p-1} frac{frac{(p-1)!}{k}}{(p-1)!}$$



      Now $frac{(p-1)!}{k}$ is an integer for each such $k$, so writing $A=(p-1)!$, we note that $frac{A}{k} equiv_p (A mod p)(k^{-1} mod p)$. Thus



      $$sum_{k=1}^{p-1} frac{(p-1)!}{k} doteq sum_{k=1}^{p-1} frac{A}{k} equiv_p sum_{k=1}^{p-1} (A mod p)(k^{-1} mod p).$$



      WE then use the fact that each element $k$ has a unqiue inverse to conclude



      $$sum_{k=1}^{p-1} (A mod p)(k^{-1} mod p) equiv_p sum_{k=1}^{p-1} (A mod p)k.$$



      However, one can check that for a prime $p geq 3$ the following holds: $sum_{k=1}^{p-1} (k mod p) equiv_p 0$, concluding



      $$sum_{k=1}^{p-1} (A mod p)(k^{-1} mod p) equiv_p sum_{k=1}^{p-1} (A mod p)k equiv_p 0,$$



      which gives you what you need to show [make sure you see why, it follows because $p$ does not divide $(p-1)!$].






      share|cite|improve this answer











      $endgroup$
















        0












        0








        0





        $begingroup$

        Well



        $$sum_{k=1}^{p-1} frac{1}{k} = sum_{k=1}^{p-1} frac{frac{(p-1)!}{k}}{(p-1)!}$$



        Now $frac{(p-1)!}{k}$ is an integer for each such $k$, so writing $A=(p-1)!$, we note that $frac{A}{k} equiv_p (A mod p)(k^{-1} mod p)$. Thus



        $$sum_{k=1}^{p-1} frac{(p-1)!}{k} doteq sum_{k=1}^{p-1} frac{A}{k} equiv_p sum_{k=1}^{p-1} (A mod p)(k^{-1} mod p).$$



        WE then use the fact that each element $k$ has a unqiue inverse to conclude



        $$sum_{k=1}^{p-1} (A mod p)(k^{-1} mod p) equiv_p sum_{k=1}^{p-1} (A mod p)k.$$



        However, one can check that for a prime $p geq 3$ the following holds: $sum_{k=1}^{p-1} (k mod p) equiv_p 0$, concluding



        $$sum_{k=1}^{p-1} (A mod p)(k^{-1} mod p) equiv_p sum_{k=1}^{p-1} (A mod p)k equiv_p 0,$$



        which gives you what you need to show [make sure you see why, it follows because $p$ does not divide $(p-1)!$].






        share|cite|improve this answer











        $endgroup$



        Well



        $$sum_{k=1}^{p-1} frac{1}{k} = sum_{k=1}^{p-1} frac{frac{(p-1)!}{k}}{(p-1)!}$$



        Now $frac{(p-1)!}{k}$ is an integer for each such $k$, so writing $A=(p-1)!$, we note that $frac{A}{k} equiv_p (A mod p)(k^{-1} mod p)$. Thus



        $$sum_{k=1}^{p-1} frac{(p-1)!}{k} doteq sum_{k=1}^{p-1} frac{A}{k} equiv_p sum_{k=1}^{p-1} (A mod p)(k^{-1} mod p).$$



        WE then use the fact that each element $k$ has a unqiue inverse to conclude



        $$sum_{k=1}^{p-1} (A mod p)(k^{-1} mod p) equiv_p sum_{k=1}^{p-1} (A mod p)k.$$



        However, one can check that for a prime $p geq 3$ the following holds: $sum_{k=1}^{p-1} (k mod p) equiv_p 0$, concluding



        $$sum_{k=1}^{p-1} (A mod p)(k^{-1} mod p) equiv_p sum_{k=1}^{p-1} (A mod p)k equiv_p 0,$$



        which gives you what you need to show [make sure you see why, it follows because $p$ does not divide $(p-1)!$].







        share|cite|improve this answer














        share|cite|improve this answer



        share|cite|improve this answer








        edited Jan 2 at 17:57

























        answered Jan 2 at 17:29









        MikeMike

        4,790512




        4,790512























            0












            $begingroup$

            For $kleq {p-1over 2}$ let $q_k := {1over k(p-k)}$



            So we have $$({1over 1}+{1over p-1})+({1over 2}+{1over p-2})+...+({1over {p-1over 2}}+{1over {p+1over 2}}) = {aover b}$$



            $$pq_1+pq_2+...+pq_{p-1over 2} = {aover b}$$



            Let $$q_1+q_2+...+q_{p-1over 2} = {cover (p-1)!}$$ for some integer $c$. So we have $$pcdot c cdot b = acdot (p-1)!implies pmid acdot (p-1)!implies pmid a$$






            share|cite|improve this answer









            $endgroup$













            • $begingroup$
              Is this not enough answer to your question?
              $endgroup$
              – Maria Mazur
              Jan 8 at 18:46
















            0












            $begingroup$

            For $kleq {p-1over 2}$ let $q_k := {1over k(p-k)}$



            So we have $$({1over 1}+{1over p-1})+({1over 2}+{1over p-2})+...+({1over {p-1over 2}}+{1over {p+1over 2}}) = {aover b}$$



            $$pq_1+pq_2+...+pq_{p-1over 2} = {aover b}$$



            Let $$q_1+q_2+...+q_{p-1over 2} = {cover (p-1)!}$$ for some integer $c$. So we have $$pcdot c cdot b = acdot (p-1)!implies pmid acdot (p-1)!implies pmid a$$






            share|cite|improve this answer









            $endgroup$













            • $begingroup$
              Is this not enough answer to your question?
              $endgroup$
              – Maria Mazur
              Jan 8 at 18:46














            0












            0








            0





            $begingroup$

            For $kleq {p-1over 2}$ let $q_k := {1over k(p-k)}$



            So we have $$({1over 1}+{1over p-1})+({1over 2}+{1over p-2})+...+({1over {p-1over 2}}+{1over {p+1over 2}}) = {aover b}$$



            $$pq_1+pq_2+...+pq_{p-1over 2} = {aover b}$$



            Let $$q_1+q_2+...+q_{p-1over 2} = {cover (p-1)!}$$ for some integer $c$. So we have $$pcdot c cdot b = acdot (p-1)!implies pmid acdot (p-1)!implies pmid a$$






            share|cite|improve this answer









            $endgroup$



            For $kleq {p-1over 2}$ let $q_k := {1over k(p-k)}$



            So we have $$({1over 1}+{1over p-1})+({1over 2}+{1over p-2})+...+({1over {p-1over 2}}+{1over {p+1over 2}}) = {aover b}$$



            $$pq_1+pq_2+...+pq_{p-1over 2} = {aover b}$$



            Let $$q_1+q_2+...+q_{p-1over 2} = {cover (p-1)!}$$ for some integer $c$. So we have $$pcdot c cdot b = acdot (p-1)!implies pmid acdot (p-1)!implies pmid a$$







            share|cite|improve this answer












            share|cite|improve this answer



            share|cite|improve this answer










            answered Jan 2 at 18:10









            Maria MazurMaria Mazur

            50.5k1361126




            50.5k1361126












            • $begingroup$
              Is this not enough answer to your question?
              $endgroup$
              – Maria Mazur
              Jan 8 at 18:46


















            • $begingroup$
              Is this not enough answer to your question?
              $endgroup$
              – Maria Mazur
              Jan 8 at 18:46
















            $begingroup$
            Is this not enough answer to your question?
            $endgroup$
            – Maria Mazur
            Jan 8 at 18:46




            $begingroup$
            Is this not enough answer to your question?
            $endgroup$
            – Maria Mazur
            Jan 8 at 18:46



            Popular posts from this blog

            Biblatex bibliography style without URLs when DOI exists (in Overleaf with Zotero bibliography)

            ComboBox Display Member on multiple fields

            Is it possible to collect Nectar points via Trainline?